LSAT and Law School Admissions Forum

Get expert LSAT preparation and law school admissions advice from PowerScore Test Preparation.

 Administrator
PowerScore Staff
  • PowerScore Staff
  • Posts: 8917
  • Joined: Feb 02, 2011
|
#63906
Complete Question Explanation

Must Be True. The correct answer choice is (E)

This stimulus presents a comparison between the costs associated with fiber-optic cable and those
associated with copper telephone cable. The fiber optic kind is more expensive to manufacture,
yet it is the more cost-effective choice. The reason provided is that copper cable requires frequent
amplification to carry complex electrical signals over long distances. The fiber optic option, on the
other hand, carries pulses of light, which can travel much farther before amplification is needed.

In sum, the fiber optic cables cost more to make, but they are more cost effective because they
require less signal amplification. Clearly, if fiber optic cable is more cost effective overall, the cost
savings on amplification must be more than enough to compensate for the extra manufacturing costs.
The question stem which follows is a Must Be True, which means that the correct answer choice
must pass the Fact Test, and can be confirmed by the information from the stimulus alone.

Answer choice (A): Just because fiber-optic cable is more expensive to manufacture than copper
cable does not necessarily mean that the material from which it is made is more expensive than
copper. It is possible, for example, that the added cost of making fiber-optic cable is due to the
equipment needed to make it, or to the complexity of the manufacturing process itself.

Answer choice (B): Although the stimulus provides that copper requires more amplification to travel
the same distance, the author provides no evidence that telephone networks that still use copper
cable are somehow strained. Remember: you cannot bring in information from outside the stimulus
to answer the questions, even if such information is true in the “real” world. All of the information
necessary to answer the question resides in the stimulus.

Answer choice (C): The author does not compare the number of signals that can be carried at
once, by fiber-optic vs. copper cable. As with Answer choice (B), be careful not to bring in outside
information when answering a Must Be True question. The correct answer choice must be confirmed
by the information presented in the stimulus.

Answer choice (D): Because the relative speed of transmission of fiber-optic vs. copper cable is
never discussed in the stimulus, this answer choice is incorrect.

Answer choice (E): This is the correct answer choice. Even if this choice had not been prephrased,
we could confirm it as the right answer with the Fact Test: It must be true that the cost savings with
fiber optic cable outweigh the extra costs associated with its manufacture.
 Tuothekhazar
  • Posts: 20
  • Joined: May 28, 2020
|
#77887
P( TC/F ) > P ( TC/ C ) = Price of producing ( Telephone Cable / Fiber Optic ) > It of producing ( Telephone Cable / Copper )

A*X > A* X2

P( TC/FT ) < P ( TC/ CT ) = Price associated with ( Telephone Cable / Fiber Optic signal Travels ) < it with ( Telephone Cable / Copper signal Travels )

A*Y < A* Y2

* Inference: if premise 2 guarantees the conclusion, then we know no any other costs occurs from any possible items network requires between Fiber and Coper should be considered.

C: ALL P of { P( TC/F ) + P( TC/FT ) } < ALL P { P( TC/C ) + P( TC/ CT ) }

A *( X + Y ) < A* ( X2 + Y2 ) = ( AX - AX2 ) < ( AY2 - AY )

Predict Answer:


a. Necessary Assumption: So, IF A ( X + Y ) always < A ( X2 + Y2 ), then ( AY2 - AY1 ) will always > ( AX - AX2 )
b. Sufficient Assumption: AY2 always > A{ ( X- X2) + Y }

The above statements, if true, most strongly support which one of the following?

The material from which fiber-optic cable is manufactured is more expensive than the copper from which copper cable is made.

Premise Booster

(B) The increase in the number of transmissions of complex signals through telephone cables is straining those telephone networks that still use copper cable.

It actually weaken the argument, since if its the “damage “ causes the cost increased, we can’t make any solid comparison.

(C) Fiber-optic cable can carry many more signals simultaneously than copper cable can.

It also weaken the argument by indicating it's not because the signal can travel father, but because more signals it carries to reduced the cost on amplifications.

(D) Signals transmitted through fiber-optic cable travel at the same speed as signals transmitted through copper cable.

It also weakens the argument, since S = D/T ( Speed = time/ distance ), and if they all travel the same speed, the it must be true that farther one travels, more times one needs. Unless argument indicates time differences does not cause any cost increase, we can’t infer anything based on the answer.

(E) The cost associated with frequent amplification of signals traveling through copper cable exceeds the extra manufacturing cost of fiber-optic cable.

P( TC/ CT ) > P( TC/ F - TC/ C ) = AY2 > A( X - X2 ), is it matching our prediction A & B ? Yes, it is. We know that there is no any negative number be assumed in this argument, and if every number associated with costs are all positive, then If AY2 > A( X - X2 + Y ), then it must be true that AY2 must always > ( X - X2 )
 Jeremy Press
PowerScore Staff
  • PowerScore Staff
  • Posts: 1000
  • Joined: Jun 12, 2017
|
#78604
Hi Tuo,

Just a quick comment on your answer-choice analysis here. This is a "most strongly supported" form of Must Be True question, and, as such, we're not evaluating answer choices based on what effect they have on the stimulus. Rather, we're evaluating them based on whether they are supported by the stimulus (i.e. whether they follow logically from the statements in the stimulus). So, just be careful about rejecting an answer as something that "weakens" the stimulus in a Must Be True question (as you have with answers B, C, and D). Rather, as the initial explanation does, ask whether the statements in the stimulus "support" those answers. For the reasons the initial explanation helpfully lays out, those answers cannot be supported from the statements given in the stimulus.

I hope this helps!

Get the most out of your LSAT Prep Plus subscription.

Analyze and track your performance with our Testing and Analytics Package.